Ongoing and Integrated - Saginaw Valley State · Web viewOngoing and Integrated B1.1a Generate...

122
BIOLOGY ASSESSMENTS Ongoing and Integrated B1.1a Generate new questions that can be investigated in the laboratory or field. 1. Which question could be resolved by undertaking a simple scientific investigation? A. Should a river in Tasmania be damned? B. Did birds evolve from reptiles? C. What color light assists growth of broad beans? D. Should the money for cancer research be increased? Answer: C 2. “Purple alien snails” are an invasive species to the Great Lakes region forested systems that has increased in abundance for the last few years. The Nature Conservancy is concerned about the impact of the “Purple alien snails” on several species of native snails. The Nature Conservancy claims that these “Purple alien snails” are causing a decrease in native snail diversity and abundance. Before the DNR can act on this claim, they must do an experiment to check if the “Purple alien snails” are really affecting the native snails in this way. What investigation needs to first be completed? A. Where did the purple alien snails originally come from? B. How are the purple alien snails affecting native snail species? C. What causes the new snails to be purple? D. Where are the native snails moving? Answer: B Biology Assessment – April 2008 1

Transcript of Ongoing and Integrated - Saginaw Valley State · Web viewOngoing and Integrated B1.1a Generate...

Page 1: Ongoing and Integrated - Saginaw Valley State · Web viewOngoing and Integrated B1.1a Generate new questions that can be investigated in the laboratory or field. 1. Which question

BIOLOGY ASSESSMENTS

Ongoing and Integrated

B1.1a Generate new questions that can be investigated in the laboratory or field.

1. Which question could be resolved by undertaking a simple scientific investigation?

A. Should a river in Tasmania be damned?B. Did birds evolve from reptiles?C. What color light assists growth of broad beans?D. Should the money for cancer research be increased?

Answer: C

2. “Purple alien snails” are an invasive species to the Great Lakes region forested systems that has increased in abundance for the last few years. The Nature Conservancy is concerned about the impact of the “Purple alien snails” on several species of native snails. The Nature Conservancy claims that these “Purple alien snails” are causing a decrease in native snail diversity and abundance. Before the DNR can act on this claim, they must do an experiment to check if the “Purple alien snails” are really affecting the native snails in this way.  What investigation needs to first be completed?

A. Where did the purple alien snails originally come from?B. How are the purple alien snails affecting native snail species?C. What causes the new snails to be purple?D. Where are the native snails moving?

Answer: B

B1.1b Evaluate the uncertainties or validity of scientific conclusions using an understanding of sources of measurement error, the challenges of controlling variables, accuracy of data analysis, logic of argument, logic of experimental design, and/or the dependence on underlying assumptions.

1. A medical researcher hypothesizes that a newly developed medication can reduce high blood pressure. Which of these would most likely be the dependent variable in a study involving this medication?

A. The number of participants in the study B. The ages of people treated for high blood pressure with other medications C. The blood pressure of the participants in the study D. The number of people treated for high blood pressure with other medications

Answer: C

Biology Assessment – April 2008 1

Page 2: Ongoing and Integrated - Saginaw Valley State · Web viewOngoing and Integrated B1.1a Generate new questions that can be investigated in the laboratory or field. 1. Which question

Sample Passage 1: Biology, Data Representation Format

A scientist investigated the factors that affect seed mass in the plan species Desnodium poniculatum. Some results of this study are summarized in the two tables below.

Table 1

Daylight hours Other variableAverage seed mass (in mg) of plants raised at:

23C 29C

14 7.10 5.63

14 Leaves removed 7.15 6.11

14 Reduced water 4.81 5.81

8 6.12

Table 2

A. Number of seeds per fruit Average seed mass (mg)12345

6.626.285.976.005.59

B. Position of seed in fruit* Average seed mass (mg)1 (closest to stem)

234

5 (farthest from stem)

5.986.065.965.825.27

*Seeds closest to the stem mature first and are released first.

2. The data suggest that subjecting plants to which of the following conditions would result in the greatest seed masses?

A. 8 hours of light, adequate water supply, and 23CB. 8 hours of light, decreased water supply, and 23CC. 14 hours of light, adequate water supply, and 23CD. 14 hours of light, decreased water supply, and 29C

Answer: C

Biology Assessment – April 2008 2

Page 3: Ongoing and Integrated - Saginaw Valley State · Web viewOngoing and Integrated B1.1a Generate new questions that can be investigated in the laboratory or field. 1. Which question

3. Which of the following conclusions is NOT consistent with the data presented in Table 2?

A. The last seed released from the plant will have a greater mass than the first seed released.

B. The first seed released from the plant will have a greater mass than the last seed released.

C. The last seed released from the plant's fruit is the farthest from the stem.D. Seeds of the smallest mass are located farthest from the plant's stem.

Answer: A

4. Suppose some of the plants in the study had been exposed to 8 hours of sunlight and a temperature of 29C. If no other variables were introduced, which of the following would be the most reasonable prediction of the average mass of the seed(s) produced under those circumstances?

A. 8.30 mgB. 7.10 mgC. 6.50 mD. 4.85 mg

Answer: D

5. Two students were testing the amount of fertilizer that would best promote the growth of strawberries in a garden. Which of the following could be an unavoidable source of experimental error?

A. length of the studyB. variation in the strawberry plantsC. the cost of watering the plantsD. fertilization during the study

Answer: B

Biology Assessment – April 2008 3

Page 4: Ongoing and Integrated - Saginaw Valley State · Web viewOngoing and Integrated B1.1a Generate new questions that can be investigated in the laboratory or field. 1. Which question

B1.1c Conduct scientific investigations using appropriate tools and techniques (e.g., selecting an instrument that measures the desired quantity—length, volume, weight, time interval, temperature—with the appropriate level of precision).

1. Supplies appropriate for the measurement in a school laboratory of the density of a small rock sample include all of the following EXCEPT

A. waterB. a graduated cylinderC. a platform balanceD. a thermometer

Answer: D

2. Suppose that a mixture of 8 g of sugar, 5.20 g of salt, and 100.01 g of fl our is prepared. What is the total mass of the mixture expressed in exponential notation and with the correct number of significant figures?

A. 1 X 102

B. 1.1 X 102

C. 1.13 X 102

D. none of the above

Answer: D

B1.1d Identify patterns in data and relate them to theoretical models.

1. Which statement about scientific models is correct?

A. A commonly accepted model can never be changed.B. Measurements which vary from a model are wrong.C. A model is rarely based on observations and knowledge.D. A model can be used to make predictions.

Answer: D

Biology Assessment – April 2008 4

Page 5: Ongoing and Integrated - Saginaw Valley State · Web viewOngoing and Integrated B1.1a Generate new questions that can be investigated in the laboratory or field. 1. Which question

2. Which of these probably served as the most useful data after Pasteur injected a group of chickens with bacteria from the older culture?

A. The health and behavior of the injected chickens B. The total number of cholera cases observed C. The changes in the shape and color of the bacterial colonies D. The recovery rate of chickens previously infected with cholera

Answer: A

B1.1e Describe a reason for a given conclusion using evidence from an investigation.

1. Which of the following are least likely to be used by astronomers to investigate galaxies?

A. Light wavesB. Radio wavesC. Sound wavesD. X-rays

Answer: C

Biology Assessment – April 2008 5

Louis Pasteur’s Experiment

Like many other scientific breakthroughs, the discovery of immunization happened by accident.

In 1880 Louis Pasteur was trying to protect chickens from cholera. To study the disease, Pasteur and his assistants gave injections of cholera bacteria to several groups of chickens. The chickens usually died from cholera after being injected with the bacteria. However, one group of chickens survived. Pasteur noticed that these chickens had been injected with a six-week-old batch of cholera bacteria.

Pasteur’s assistants prepared new colonies of cholera bacteria and injected these chickens a second time. They also injected a group of chickens that had not been previously injected. Pasteur was surprised that the chickens that had been injected previously with the old bacteria remained healthy, while all the chickens in the second group became sick and died.

This is how Pasteur came to discover that exposure to weakened pathogens can provide immunity from disease. The idea was later used to protect animals from other serious diseases. Today immunizations help protect people from many illnesses.

Page 6: Ongoing and Integrated - Saginaw Valley State · Web viewOngoing and Integrated B1.1a Generate new questions that can be investigated in the laboratory or field. 1. Which question

2. The bacteria Bacillus thuringiensis produce protein crystals that are toxic to the digestive system of insects. How can these bacteria be used to control insects in crops?

A. Apply the bacteria to growing plants. B. Expose the bacteria to low levels of light. C. Remove plants from areas containing the bacteria. D. Treat the bacteria with a solution of the protein crystals.

Answer: A

Bl.1f Predict what would happen if the variables, methods, or timing of an investigation were changed.

B1.1g Based on empirical evidence, explain and critique the reasoning used to draw a scientific conclusion or explanation.

1. When a pesticide is submitted to the United States Department of Agriculture for registration, the conditions of use must be clearly specified, along with data indicating the amount of residue that will remain after the pesticide has been applied as directed. Which of the following statements best explains why the required information may not fully reveal the environmental hazards associated with the use of the pesticide?

A. The costs in time and money required to gather accurate data are high.B. The chemical structure of the pesticide may not be known.C. The side effects associated with applications of the pesticide may alter the results.D. The conditions under which the pesticide will be used cannot be controlled.

Answer: D

Biology Assessment – April 2008 6

Page 7: Ongoing and Integrated - Saginaw Valley State · Web viewOngoing and Integrated B1.1a Generate new questions that can be investigated in the laboratory or field. 1. Which question

B1.1h Design and conduct a systematic scientific investigation that tests a hypothesis. Draw conclusions from data presented in charts or tables.

1. The table shows the time it took trees of the same type and size to drop all of their leaves after being grown in different atmospheres. For the experiment to be valid, which of these must be a constant for all the trees?

A. The height of all the trees during the entire experiment B. The amount of water available to all trees during the experiment C. The mass of fallen leaves collected from each tree D. The rate of photosynthesis in all trees during the experiment

Answer: B

2. The graph below represents data gathered during an experiment on cellular respiration.

Which of the following conclusions is best supported by data from this graph?

A. ATP production is independent of sugar availability.

B. The amount of cellular respiration is constant in muscle cells.

C. ATP is only produced when sugar concentrations are above 4 g/L.

D. The amount of cellular respiration increases as sugar concentration increases.

Answer: DB1.1i Distinguish between scientific explanations that are regarded as current scientific consensus and the emerging questions that active researchers investigate.

Biology Assessment – April 2008 7

Page 8: Ongoing and Integrated - Saginaw Valley State · Web viewOngoing and Integrated B1.1a Generate new questions that can be investigated in the laboratory or field. 1. Which question

B1.2a Critique whether or not specific questions can be answered through scientific investigations.

B1.2b Identify and critique arguments about personal or societal issues based on scientific evidence.

B1.2c Develop an understanding of a scientific concept by accessing information from multiple sources. Evaluate the scientific accuracy and significance of the information.

1. A scientist reported a new discovery. This discovery did not agree with the existing scientific theory, and caused controversy. What would be the best for the scientific community to do?

A. Ignore the discovery that has been reportedB. Reject the existing scientific theoryC. Have other scientists repeat the scientist’s experimentsD. Survey other people about the discovery

Answer: C

Read the investigation below and use the information provided to answer question 2.

Hypothesis – If mutations can affect plant growth, then exposing plant seeds to x-rays will affect plant growth.

Materials – Dental x-ray machine, 4 flower pots, 4 pepper plant seeds, Water, Labels, Soil

Method – Fill each pot with soil and place a pepper plant seed on the surface. Expose three of the four pots to the x-ray machine for different lengths of time, one for 2.5 seconds, one for 5 seconds and one for 10 seconds. Label each pot according to how long it was exposed to the x-ray machine. Label the unexposed pot “0 seconds”. Plant each seed and give each pot equal water and sunlight. Record the growth of each plant every two weeks for six weeks.

Biology Assessment – April 2008 8

Page 9: Ongoing and Integrated - Saginaw Valley State · Web viewOngoing and Integrated B1.1a Generate new questions that can be investigated in the laboratory or field. 1. Which question

Results – After six weeks, the investigators produced the following table:

PEPPER PLANT GROWTH TABLEX-RAY

EXPOSURE TIMEGrowth at 2

weeksGrowth at 4

weeksGrowth at 6

weeks0 seconds 3.2 cm 10.7 cm 18.8 cm

2.5 seconds 1.7 cm 5.7 cm 12.5 cm5 seconds 3.3 cm 11.0 cm 19.3 cm10 seconds 3.1 cm 10.4 cm 18.1 cm

2. The investigators assume x-rays can do which of the following?

A. prevent meiosisB. destroy antibodiesC. alter DNA moleculesD. inhibit natural selection

Answer: C

B1.2d Evaluate scientific explanations in a peer review process or discussion format.

B1.2e Evaluate the future career and occupational prospects of science fields.

B1.2f Critique solutions to problems, given criteria and scientific constraints.

B1.2g Identify scientific tradeoffs in design decisions and choose among alternative solutions.

1. A computer model of cellular mitosis can simulate the aspects of cellular division quite well. However, microscopic observation of actual cellular mitosis can improve understanding because actual observations

A. may reveal greater unknown complexitiesB. are easier than a computer model to viewC. are the same each timeD. may provide division events in sequence

Answer: A or D

Biology Assessment – April 2008 9

Page 10: Ongoing and Integrated - Saginaw Valley State · Web viewOngoing and Integrated B1.1a Generate new questions that can be investigated in the laboratory or field. 1. Which question

B1.2h Describe the distinctions between scientific theories, laws, hypotheses, and observations.

1. In most stable freshwater environments, populations of Daphnia are almost entirely female and reproduce asexually. However, males are observed in low oxygen environments or when food is scarce. Based on these observations, a researcher suggests that male Daphnia develop in response to unfavorable environmental conditions. This is an example of a

A. resultB. theoryC. procedureD. hypotheses

Answer: D

2. Which of the following meanings of "theory" best fits the meaning scientists use when they refer to "Cell Theory" or "Theory of Evolution"?

A. Possible but untested ideas.B. An idea someone has about how or when or why something happens.C. A fact or bit of evidence available to explain something.D. A set of tested positions, explanations and concepts that explain known data and

predict additional phenomena.

Answer: D

Bl.2i Explain the progression of ideas and explanations that lead to science theories that are part of the current scientific consensus or core knowledge.

B1.2j Apply science principles or scientific data to anticipate effects of technological design decisions

1. A researcher was trying to determine the effects of the drug AZT on the HIV virus, using 200 infected rats. One hundred of the rats were each given a daily injection of 10 ml of a solution of AZT and water for a period of 3 months. What should the researcher give the remaining 100 rats each day?

A. an injection of 10 ml of AZTB. an injection of 10 ml of waterC. an oral dose of 10 ml of waterD. food containing 10 ml of AZT

Answer: B

Biology Assessment – April 2008 10

Page 11: Ongoing and Integrated - Saginaw Valley State · Web viewOngoing and Integrated B1.1a Generate new questions that can be investigated in the laboratory or field. 1. Which question

B1.2k Analyze how science and society interact from a historical, political, economic, or social perspective.

1. Which occupation would be LEAST concerned with the biomass production in a given county?

A. OptometryB. Agricultural scienceC. EcologyD. Soil science

Answer: A

2. Which of these questions would an ornithologist (a person who studies birds) ask an ethologist (a person who studies animal behavior) for help in answering?

A. Why do some birds avoid viceroy butterflies?B. How do birds metabolize sugars?C. Which adaptations enable a bird to fly?D. Which birds are best suited for cold environments?

Answer: A

Biology Assessment – April 2008 11

Page 12: Ongoing and Integrated - Saginaw Valley State · Web viewOngoing and Integrated B1.1a Generate new questions that can be investigated in the laboratory or field. 1. Which question

Unit 1: Chemistry and BiochemistryTime: 2.5 weeks

B2.2 Organic Molecules There are four major categories of organic molecules that make up living systems: carbohydrates, fats, proteins, and nucleic acids.

1.  What property of water is most important for living organisms? 

A.  It is odorless.B.  It does not conduct electricity.C.  It is tasteless.D.  It is liquid at most temperatures on Earth.

Answer: D

B2.2a Explain how carbon can join to other carbon atoms in chains and rings to form large and complex molecules.

1. Which of the following is the best explanation of why carbon is the building block of all organic molecules?

A. Carbon can bond with itself to form chains and rings. B. Carbon can form 4 different bonds with many different elements. C. Carbon is the most abundant element on earth. D. Both A and B are correct.

Answer: D

B2.2b Recognize the six most common elements in organic molecules.

1. When digging at an archeological site in Egypt a scientist comes across a large gold box. They quickly open it to see if it holds any information about the ancient Egyptian life they have been studying. The material inside is unrecognizable in its current state. The scientist thinks it may be organic remains. What elements should they test the material for to determine if it is organic?

A. carbon, nitrogen, oxygen, phosphorus, hydrogen, sulfurB. phosphate, nitrogen, carbonC. carbon, nitrogen, helium, phosphate, oxygen, sodiumD. calcium, nitrogen, oxygen, phosphate, helium, sodium

Answer: A

Biology Assessment – April 2008 12

Page 13: Ongoing and Integrated - Saginaw Valley State · Web viewOngoing and Integrated B1.1a Generate new questions that can be investigated in the laboratory or field. 1. Which question

B2.2c Describe the composition of the four major categories of organic molecules (carbohydrates, lipids, proteins, and nucleic acids).

B2.2d Explain the general structure and primary functions of the major complex organic molecules that compose living organisms.

1. Which of the following describes the function of proteins?

A. energy formation and storageB. energy used in muscles and reactionC. structural use and enzyme formationD. heredity and genetic code carriers

Answer: C

2. Fats are important energy storage compounds because they:

A. readily breakdown to form glucose. B. provide immediate energy. C. contain more energy per gram than carbohydrates or proteins. D. are liquid at normal body temperatures.

Answer: C

B2.2e Describe how dehydration and hydrolysis relate to organic molecules.

1. Many land plants store energy in starch. When energy is needed, the starch molecules can be broken down quickly. The chemical reaction produces which of the following?

A. amino acidsB. lipidsC. monosaccharidesD. RNA chains

Answer: C

B2.2x Proteins Protein molecules are long, usually folded chains composed mostly of amino acids and are made of C, H, O, and N. Protein molecules assemble fats and carbohydrates; they function as enzymes, structural components, and hormones. The function of each protein molecule depends on its specific sequence of amino acids and the shape of the molecule.

Biology Assessment – April 2008 13

Page 14: Ongoing and Integrated - Saginaw Valley State · Web viewOngoing and Integrated B1.1a Generate new questions that can be investigated in the laboratory or field. 1. Which question

1. According to Figure 2, Enzyme A has the fastest rate of reaction at a pH closest to:         

A. 5B. 6C. 8D. 9

Answer: A

Biology Assessment – April 2008 14

Page 15: Ongoing and Integrated - Saginaw Valley State · Web viewOngoing and Integrated B1.1a Generate new questions that can be investigated in the laboratory or field. 1. Which question

2. Based on the data in Figure 2, at which of the following pHs, if any, do Enzymes A and B have the same acceleration factor?

A. At pH 5 onlyB. At pH 6.7 onlyC. At all pHs between 2 and 11D. At none of the pHs shown in the fig

Answer: C

3. A scientist claims that the acceleration factor of Enzyme B is dependent on both enzyme and substrate concentration. Do the data in Figures 3 and 4 support her claim?

A. No; the acceleration factor is dependent on enzyme concentration, but not on substrate concentration.

B. No; the acceleration factor is not dependent on either enzyme or substrate concentration.

C. Yes; the acceleration factor is dependent on enzyme concentration, but not on substrate concentration.

D. Yes; the acceleration factor is dependent on both enzyme and substrate concentration.

Answer: D

4. A scientist claims that for the conditions used to obtain the data for Figure 4, the acceleration factor of Enzyme B at a given concentration will always be greater than that of Enzyme A at the same concentration. Do the data support his conclusion?

A. No; Enzyme B shows a lower acceleration factor at all the enzyme concentrations tested.

B. No; Enzyme B shows a lower acceleration factor at all the substrate concentrations tested.

C. Yes; Enzyme B shows a higher acceleration factor at all the enzyme concentrations tested

D. Yes; Enzyme B shows a higher acceleration factor at all the substrate concentrations tested.

Answer: A

Biology Assessment – April 2008 15

Page 16: Ongoing and Integrated - Saginaw Valley State · Web viewOngoing and Integrated B1.1a Generate new questions that can be investigated in the laboratory or field. 1. Which question

B2.2f Explain the role of enzymes and other proteins in biochemical functions (e.g., the protein hemoglobin carries oxygen in some organisms, digestive enzymes and hormones).

1. In red blood cells, the compound carbonic anhydrase increases the rate at which carbon dioxide is converted to bicarbonate ions for transport in the blood. IN red blood cells, carbonic anhydrase acts as which of the following?

A. an enzymeB. a hormoneC. a lipidD. a sugar

Answer: A

B2.4 Cell Specialization In multicellular organisms, specialized cells perform specialized functions. Organs and organ systems are composed of cells and function to serve the needs of cells for food, air, and waste removal. The way in which cells function is similar in all living organisms.

B2.4f Recognize and describe that both living and nonliving things are composed of compounds, which are themselves made up of elements joined by energy containing bonds, such as those in ATP.

1. Describe how living, non-living, compounds, and elements are connected.

A. Both living and non-living things are made up of compounds. The compounds are made up of different combinations of elements.

B. Living things are made up of compounds. Non-living things are made up of elements.

C. Elements are made up of compounds which make up non-living things. The non-living things made up living things.

D. None of the above.

Answer: A

B2.5a Recognize and explain that macromolecules such as lipids contain high energy bonds.

1. Lipids are useful to humans as energy storing molecules. Which of the following is the best explanation of why this is so?

A. Lipids are great for conserving heat energy. B. Lipids contain a large amount of carbon to carbon and carbon to hydrogen bonds.

These bonds, when broken, can release large amounts of energy. C. Lipids are found in cell membranes, where energy is stored. D. None of the above.

Answer: B

Biology Assessment – April 2008 16

Page 17: Ongoing and Integrated - Saginaw Valley State · Web viewOngoing and Integrated B1.1a Generate new questions that can be investigated in the laboratory or field. 1. Which question

B2.5 Living Organism Composition All living or once-living organisms are composed of carbohydrates, lipids, proteins, and nucleic acids. Carbohydrates and lipids contain many carbon-hydrogen bonds that also store energy.

Biology Assessment – April 2008 17

Page 18: Ongoing and Integrated - Saginaw Valley State · Web viewOngoing and Integrated B1.1a Generate new questions that can be investigated in the laboratory or field. 1. Which question

Unit 2: Cells: Structure and Function Time: 2 weeks

B2.4 Cell Specialization In multicellular organisms, specialized cells perform specialized functions. Organs and organ systems are composed of cells and function to serve the needs of cells for food, air, and waste removal. The way in which cells function is similar in all living organisms.

B2.4g Explain that some structures in the modern eukaryotic cell developed from early prokaryotes, such as mitochondria, and in plants, chloroplasts.

1. Blue green algae and chloroplasts look remarkably similar under a microscope. Also, molecular geneticists have found that their DNA is also very similar. Given the fact that bacteria and single celled eukaryotes often ingest one another, which is the best explanation of this phenomenon?

A. Early eukaryotes ingested bacteria similar to blue green algae. Instead of digesting them, the blue green algae provided a mutualistic relationship.

B. This is just a remarkable coincidence. C. All things are made up of DNA, so naturally things will look similar and have

similar DNA. D. None of the above.

Answer: D

B2.4i Recognize that while viruses lack cellular structure, they have the genetic material to invade living cells.

1. Which of these does a virus need in order to multiply?

A. Chloroplasts from a host cell B. A host cell to provide oxygen for the virus C. New ADP from a host cell D. A host cell to replicate the virus’s DNA

Answer: D

Biology Assessment – April 2008 18

Page 19: Ongoing and Integrated - Saginaw Valley State · Web viewOngoing and Integrated B1.1a Generate new questions that can be investigated in the laboratory or field. 1. Which question

B2.4h Describe the structures of viruses and bacteria.

1. Which of the following is found in both cells and viruses?

A. Silica B. Genetic material C. Digestive cavity D. Flagella

Answer: B

2. Which of the following require a host cell because they are not able to make proteins on their own?

A. blue-green algaeB. bacteriaC. protozoansD. viruses

Answer: D

B2.5 Living Organism Composition All living or once-living organisms are composed of carbohydrates, lipids, proteins, and nucleic acids. Carbohydrates and lipids contain many carbon-hydrogen bonds that also store energy.

B2.5g Compare and contrast plant and animal cells.

1. Which is a characteristic of members of the plant kingdom that distinguishes them from members of the animal kingdom?

A. Storage of energy in chemical bonds B. Exchange of H2O with the environment C. Use of mRNA during protein production D. Use of chlorophyll for solar-energy transformation

Answer: D

2.  Viet and Andrea were using a microscope to look at a slide of some cells. They looked at some interesting cells that Viet thought were plant cells. Andrea thought they looked more like animal cells. If you looked at these same cells, how could you tell whether they were plant cells or animal cells?

Biology Assessment – April 2008 19

Page 20: Ongoing and Integrated - Saginaw Valley State · Web viewOngoing and Integrated B1.1a Generate new questions that can be investigated in the laboratory or field. 1. Which question

B2.5h Explain the role of cell membranes as a highly selective barrier (diffusion, osmosis, and active transport).

1. Which of these is a function of the cell membrane in all cells?

A. Producing cellular nutrients B. Preserving cellular wastes C. Neutralizing chemicals D. Maintaining homeostasis

Answer: D

2. Which of these best explains why a freshwater aquarium would be a dangerous habitat for saltwater fish?

A. The tissues of the saltwater fish would absorb too much acid. B. The organs of the saltwater fish would produce too much protein. C. The organ systems of the saltwater fish would consume too much energy. D. The cells of the saltwater fish would gain too much water.

Answer: C

3.  A farmer thinks that the vegetables on her farm are not getting enough water. Her son suggests that they use water from the nearby ocean to water the vegetables. Is this a good idea?

  A.  Yes, because there is plenty of ocean water.B.  Yes, because ocean water has many natural fertilizers.C.  No, because ocean water is too salty for plants grown on land.D.  No, because ocean water is much more polluted than rainwater.

Answer: C

Biology Assessment – April 2008 20

Page 21: Ongoing and Integrated - Saginaw Valley State · Web viewOngoing and Integrated B1.1a Generate new questions that can be investigated in the laboratory or field. 1. Which question

4. Using the table below, which of the following best explains what is causing the red blood cells in solution 1 to change size over the five-minute period?

A. Solvent is entering the cells faster than it is leaving the cells.B. The cells are making new protein.C. The cell membranes are dissolving.D. All of the above.

Answer: A

Solution Time1 min. 2 min. 3 min. 4 min. 5 min.

Solution 1 No change Cells are slightly larger.

Cells are much larger.

Cells are huge.

Cells are gone.

Solution 2 No change No change No change No change No changeSolution 3 No change Cells are

slightly smaller.

Cells are much smaller.

Cells look wilted.

Nothing that looks like a cell can be found.

B2.5i Relate cell parts/organelles to their functions.

1. The diagram shows different parts of a human sperm cell. Which part of the cell is most likely specialized for mobility?

A. Q B. R C. S D. T

Answer: D

2. A cell from heart muscle would PROBABLY have an unusually high proportion of

A. lysosomesB. mitochondriaC. mRNAD. Golgie bodies

Answer: B

Biology Assessment – April 2008 21

Page 22: Ongoing and Integrated - Saginaw Valley State · Web viewOngoing and Integrated B1.1a Generate new questions that can be investigated in the laboratory or field. 1. Which question

Unit 3: Cell Energetics Time: 1 week

B2.4 Cell Specialization In multicellular organisms, specialized cells perform specialized functions. Organs and organ systems are composed of cells and function to serve the needs of cells for food, air, and waste removal. The way in which cells function is similar in all living organisms.

B2.4e Explain how cellular respiration is important for the production of ATP (build on aerobic vs. anaerobic).

1. Earthworms “breathe” through their skin. When it rains they need to come out of the soil otherwise they will die. In terms of cellular respiration, explain the process that will take place if an earthworm stays underground in a wet environment.

A. If they stay underground, they will get water in their lungs thus making it impossible for respiration to take place.

B. If they stay underground they will be taking in water rather than oxygen. With no oxygen cellular respiration can not take place thus stopping ATP production. With no ATP production cellular processes will begin to shut down. Eventually the entire organism will die.

C. If they stay underground they will be taking in water rather than oxygen. With no oxygen cellular respiration can not take place thus stopping ATP production. With no ATP production the brain will shut down first thus killing the earthworm.

D. If they stay underground they will be taking in water rather than oxygen. If water enters their lungs it will kick start anaerobic respiration. Earthworms can not stay alive on anaerobic respiration alone.

Answer: B

B2.5 Living Organism Composition All living or once-living organisms are composed of carbohydrates, lipids, proteins, and nucleic acids. Carbohydrates and lipids contain many carbon-hydrogen bonds that also store energy.

B2.5d Describe how individual cells break down energy-rich molecules to provide energy for cell functions.

1. Fats are important energy storage compounds because they:

A. readily breakdown to form glucose. B. provide immediate energy. C. contain more energy per gram than carbohydrates or proteins. D. are liquid at normal body temperatures.

Answer: C

Biology Assessment – April 2008 22

Page 23: Ongoing and Integrated - Saginaw Valley State · Web viewOngoing and Integrated B1.1a Generate new questions that can be investigated in the laboratory or field. 1. Which question

B2.5x Energy Transfer All living or once-living organisms are composed of carbohydrates, lipids, proteins, and nucleic acids. Carbohydrates and lipids contain many carbon-hydrogen bonds that also store energy. However, that energy must be transferred to ATP (adenosine triphosphate) to be usable by the cell.

B2.5e Explain the interrelated nature of photosynthesis and cellular respiration in terms of ATP synthesis and degradation.

1. Photosynthesis transforms ______________ energy to chemical energy in the form of glucose, whereas respiration transforms chemical energy in the form of glucose to cellular energy in the form of ____________.

A. ATP, light B. Light, ATPC. Food, SugarD. Carbon dioxide, Oxygen

Answer: B

B2.5f Relate plant structures and functions to the process of photosynthesis and respiration.

1. Plants have unique cellular make up in comparison to animals. In what part of a plant would you expect to find more chloroplasts?

A. stemB. rootsC. flowerD. leaf

Answer: D

B3.1 Photosynthesis and Respiration Organisms acquire their energy directly or indirectly from sunlight. Plants capture the Sun’s energy and use it to convert carbon dioxide and water to sugar and oxygen through the process of photosynthesis. Through the process of cellular respiration, animals are able to release the energy stored in the molecules produced by plants and use it for cellular processes, producing carbon dioxide and water.

Biology Assessment – April 2008 23

Page 24: Ongoing and Integrated - Saginaw Valley State · Web viewOngoing and Integrated B1.1a Generate new questions that can be investigated in the laboratory or field. 1. Which question

B3.1f Summarize the process of photosynthesis.

1. Select and put in sequence the following phenomena involved in photosynthesis: 1. production of carbon dioxide and carbohydrates 2. production of oxygen and carbohydrates 3. exposure to light 4. breakdown of proteins 5. intake of carbon dioxide 6. intake of oxygen

A. 3-4-1B. 4-3-2C. 3-5-2D. 3-6-1

Answer: C

B3.1c Recognize the equations for photosynthesis and respiration and identify the reactants and products for both (also repeated in Ecology).

1. Choose the appropriate equation for photosynthesis.

A. Carbon dioxide + water + light glucose + oxygen B. Glucose + oxygen ATP + carbon dioxide + waterC. Carbon dioxide + oxygen + light glucose + carbon dioxide D. None of the above.

Answer: A

2. Below are all the reactants and products of respiration. Identify the reactants and the products. Carbon dioxide, oxygen, ATP, glucose, water

A. Reactants = water, ATP Products = carbon dioxide, oxygen, glucose B. Reactants = oxygen, glucose Products = ATP, carbon dioxide, water C. Reactants = carbon dioxide, glucose Products = ATP, oxygen, water D. Reactants = oxygen, ATP, water Products = carbon dioxide, glucose

Answer: B

Biology Assessment – April 2008 24

Page 25: Ongoing and Integrated - Saginaw Valley State · Web viewOngoing and Integrated B1.1a Generate new questions that can be investigated in the laboratory or field. 1. Which question

B3.1b Illustrate and describe the energy conversions that occur during photosynthesis and respiration (also repeated in Ecology).

1. Kyle had a plant seed that weighed 0.5 grams and he had 25 grams of dry soil. He wanted to see how big the seed could get if he watered it every day for 2 months. He planted the seed in the soil and gave it the same amount of water each day. After 2 months the seed had grown into a plant 2 feet tall. He then dried and weighed the plant and soil again. The plant now had a mass of 8.5 grams and the soil stilled had a mass of 25 grams. Where did most of the plant’s mass come from?

A. The water provides all of the mass for the plant. B. Minerals in the soil allow the plant to gain mass. C. Carbon dioxide in the air. D. Sunlight.

Answer: C

2. Humans are continuously gaining and losing weight. Explain how, in terms of respiration, you could lose 10 pounds if you cut back calories and got on an exercise program.

A. When you exercise your cells are in need of more ATP to power cellular activity. Cellular respiration can make ATP by breaking down food molecules or fat stored in your body. If you limit your food intake then your fat storage will be used to power respiration.

B. You lose more weight when you exercise and cut back on food intake because the water weight from the food is sweated out when you’re working out.

C. Respiration is used to create food energy for your cells to use when you are working out. Therefore you need less food intake and you can lose weight.

D. None of the above.

Answer: A

Biology Assessment – April 2008 25

Page 26: Ongoing and Integrated - Saginaw Valley State · Web viewOngoing and Integrated B1.1a Generate new questions that can be investigated in the laboratory or field. 1. Which question

Unit 4: Comparative Structure and Function of Living ThingsTime: 1.5 weeks

B2.4 Cell Specialization In multicellular organisms, specialized cells perform specialized functions. Organs and organ systems are composed of cells and function to serve the needs of cells for food, air, and waste removal. The way in which cells function is similar in all living organisms.

B2.4b Describe how various organisms have developed different specializations to accomplish a particular function and yet the end result is the same (e.g., excreting nitrogenous wastes in animals, obtaining oxygen for respiration).

B2.4c Explain how different organisms accomplish the same result using different structural specializations (gills vs. lungs vs. membranes).

1. Similar structures are present in the embryos of fish, chickens, and rabbits. In fish, these structures develop into gills, but chickens and rabbits they either disappear or develop into other body parts later in embryonic development.

Which of the statements below best explains the presence of these structures in the embryos of all three species?

A. The embryos of the three species are similar in size. B. Breathing structures are similar among the young of the three species. C. The three species have a common ancestor with these embryonic structures. D. The reproductive mechanisms are similar among the adults of the three species.

Answer: C

B2.5b Explain how major systems and processes work together in animals and plants, including relationships between organelles, cells, tissues, organs, organ systems, and organisms. Relate these to molecular functions.

1. The respiratory system depends on the nervous system for signals to

A. enhance the amount of available oxygen in the lungs.B. coordinate muscles controlling breathing.C. release enzymes to increase the exchange of gases.D. exchange gases with the circulatory system.

Answer: B

B2.5 Living Organism Composition All living or once-living organisms are composed of carbohydrates, lipids, proteins, and nucleic acids. Carbohydrates and lipids contain many carbon-hydrogen bonds that also store energy.

Biology Assessment – April 2008 26

Page 27: Ongoing and Integrated - Saginaw Valley State · Web viewOngoing and Integrated B1.1a Generate new questions that can be investigated in the laboratory or field. 1. Which question

Unit 5: Human SystemsTime: 3 weeks

B2.1x Cell Differentiation Following fertilization, cell division produces a small cluster of cells that then differentiate by appearance and function to form the basic tissues of an embryo

B2.1e Predict what would happen if the cells from one part of a developing embryo were transplanted to another part of the embryo.

1. Which of the following would you predict would happen if the cells from one part of a developing embryo were transplanted to another part of the embryo?

A. The transplanted cells would take on the function of the surrounding cells. B. The surrounding cells would take on the function of the transplanted cells. C. The transplanted cells would continue with their initial function. D. The transplanted cells would die.

Answer: A

B2.3x Homeostasis The internal environment of living things must remain relatively constant. Many systems work together to maintain homeostasis. When homeostasis is lost, death occurs

B2.3d Identify the general functions of the major systems of the human body (digestion, respiration, reproduction, circulation, excretion, protection from disease, and movement, control, and coordination) and describe ways that these systems interact with one another.

1. Hemoglobin carries oxygen to body cells. Which body system contains hemoglobin?

A. Circulatory system B. Respiratory system C. Endocrine system D. Nervous system

Answer: A

Biology Assessment – April 2008 27

Page 28: Ongoing and Integrated - Saginaw Valley State · Web viewOngoing and Integrated B1.1a Generate new questions that can be investigated in the laboratory or field. 1. Which question

2. The box contains some facts about kidneys and dialysis. Which of the following best explains why dialysis works?

A. Dialysis reduces the size of

proteins. B. Proteins are dissolved by urine. C. Dialysis filters proteins from

solution. D. Proteins transport membrane

fragments

Answer: C

3.  Which pair of systems regulates and coordinates body functions? 

A.  Excretory and digestive B.  Nervous and endocrine C.  Skeletal and muscular D.  Immune and respiratory

Answer: B

4.  In your body, what two organs work together to make sure that oxygen gets to all the other organs of your body?

A.  Lungs and kidneys B.  Heart and lungs C.  Brain and kidneys D.  Heart and liver

Answer: B

5. The respiratory system depends on the nervous system for signals to A. enhance the amount of available oxygen in the lungs.B. coordinate muscles controlling breathing.C. release enzymes to increase the exchange of gases.D. exchange gases with the circulatory system.

Answer: B

Biology Assessment – April 2008 28

Proteins and other large molecules cannot move through the membranes of healthy kidneys.

The presence of large molecules in urine is an indication of unhealthy kidneys.

Dialysis is a medical treatment that does the work for failing kidneys.

In dialysis, an artificial membrane impermeable to large molecules is used.

Dialysis lowers protein levels in urine.

Kidneys and Dialysis

Page 29: Ongoing and Integrated - Saginaw Valley State · Web viewOngoing and Integrated B1.1a Generate new questions that can be investigated in the laboratory or field. 1. Which question

6. How is the excretory system most likely to respond when an animal is thirsty?

A. By relaxing the smooth muscles B. By retaining body fluids C. By absorbing heat from lymph glands D. By releasing hormones

Answer: B

B2.3g Compare the structure and function of a human body system or subsystem to a nonliving system (e.g., human joints to hinges, enzymes and substrate to interlocking puzzle pieces).

B4.3g Explain that cellular differentiation results from gene expression and/ or environmental influence (e.g., metamorphosis, nutrition).

B4.3 Cell Division- Mitosis and Meiosis Sorting and recombination of genes in sexual reproduction results in a great variety of possible gene combinations from the offspring of any two parents.

Biology Assessment – April 2008 29

Page 30: Ongoing and Integrated - Saginaw Valley State · Web viewOngoing and Integrated B1.1a Generate new questions that can be investigated in the laboratory or field. 1. Which question

Unit 6: Homeostasis and HealthTime: 2 weeks

B2.3 Maintaining Environmental Stability The internal environment of living things must remain relatively constant. Many systems work together to maintain stability. Stability is challenged by changing physical, chemical, and environmental conditions as well as the presence of disease agents.

B2.3a Describe how cells function in a narrow range of physical conditions, such as temperature and pH (acidity) to perform life functions.

1.  If air pollution causes the rain that falls on this pond to become much more acidic, after two years how will this acidity affect the living things in this pond?

  A.  There will be more plants and animals because the acid is a source of food.B.  There will be fewer plants and animals because the acid will dissolve many of

them.C.  There will be fewer plants and animals because many of them cannot survive in

water with high acidity.D.  There will be more plants and animals because the acid will kill most of the

disease-causing microorganisms.

Answer: C

B2.3b Describe how the maintenance of a relatively stable internal environment is required for the continuation of life.

1. In order for the body to maintain homeostasis, the chemical decomposition of food to produce energy must be followed by

A. water intake.B. muscle contractions.C. waste removal.D. nervous impulses.

Answer: C

B2.3c Explain how stability is challenged by changing physical, chemical, and environmental conditions as well as the presence of disease agents.

B2.3x Homeostasis The internal environment of living things must remain relatively constant. Many systems work together to maintain homeostasis. When homeostasis is lost, death occurs.

Biology Assessment – April 2008 30

Page 31: Ongoing and Integrated - Saginaw Valley State · Web viewOngoing and Integrated B1.1a Generate new questions that can be investigated in the laboratory or field. 1. Which question

B2.3e Describe how human body systems maintain relatively constant internal conditions (temperature, acidity, and blood sugar).

B2.3f Explain how human organ systems help maintain human health.

B2.6a Explain that the regulatory and behavioral responses of an organism to external stimuli occur in order to maintain both short- and long-term equilibrium.

1. Which of the following is the best example of an organism maintaining homeostasis?

A. a wolf panting after a chaseB. a spider catching an insect in a webC. a cricket becoming infected by a virusD. a mole digging tunnels in the ground

Answer: A

B2.6x Internal/External Cell Regulation Cellular processes are regulated both internally and externally by environments in which cells exist, including local environments that lead to cell differentiation during the development of multicellular organisms. During the development of complex multicellular organisms, cell differentiation is regulated through the expression of different genes.

Biology Assessment – April 2008 31

Page 32: Ongoing and Integrated - Saginaw Valley State · Web viewOngoing and Integrated B1.1a Generate new questions that can be investigated in the laboratory or field. 1. Which question

Unit 7: Matter and Energy in Ecosystems Time: 5 weeks

B2.1 Transformation of Matter and Energy in Cells In multicellular organisms, cells are specialized to carry out specific functions such as transport, reproduction, or energy transformation.

B2.1a Explain how cells transform energy (ultimately obtained from the sun) from one form to another through the processes of photosynthesis and respiration. Identify the reactants and products in the general reaction of photosynthesis

1. ATP is the form of energy required for cellular processes. Explain how your cells make ATP to carry out cellular processes?

A. Energy from the sun is captured by producers that make glucose in the process of photosynthesis. You then eat the producers and transfer glucose energy to ATP in the process of respiration.

B. Energy from the sun is captured by your cells and combined with oxygen, which allows us to make ATP.

C. Energy from the sun is captured by producers that make glucose in the process of respiration. You then eat producers and transfer glucose energy to ATP in the process of respiration.

D. You breathe in oxygen which gives you the energy to make ATP.

Answer: A

B2.1b Compare and contrast the transformation of matter and energy during photosynthesis and respiration.

1. Photosynthesis transforms ______________ energy to chemical energy in the form of glucose, whereas respiration transforms chemical energy in the form of glucose to cellular energy in the form of ____________.

A. ATP, light B. Light, ATPC. Food, SugarD. Carbon dioxide, Oxygen

Answer: B

B2.5 Living Organism Composition All living or once-living organisms are composed of carbohydrates, lipids, proteins, and nucleic acids. Carbohydrates and lipids contain many carbon-hydrogen bonds that also store energy.

Biology Assessment – April 2008 32

Page 33: Ongoing and Integrated - Saginaw Valley State · Web viewOngoing and Integrated B1.1a Generate new questions that can be investigated in the laboratory or field. 1. Which question

B2.5c Describe how energy is transferred and transformed from the Sun to energy-rich molecules during photosynthesis.

1. Light from the sun provides the energy for producers to convert carbon dioxide to glucose molecules. What process allows this transfer and why do producers make glucose molecules?

A. Cellular respiration. Glucose is a high energy molecule that can provide energy to cells.

B. Photosynthesis. Glucose provides the energy to power cellular processes. C. Photosynthesis. Glucose is an energy rich molecule that can be converted to

cellular energy. D. Cellular respiration. Glucose is a byproduct that plants give off to make oxygen.

Answer: C

B3.1 Photosynthesis and Respiration Organisms acquire their energy directly or indirectly from sunlight. Plants capture the Sun’s energy and use it to convert carbon dioxide and water to sugar and oxygen through the process of photosynthesis. Through the process of cellular respiration, animals are able to release the energy stored in the molecules produced by plants and use it for cellular processes, producing carbon dioxide and water.

B3.1a Describe how organisms acquire energy directly or indirectly from sunlight.

1. Right before Anna was about to run in a long race, she drank a large glass of orange juice to get energy. Which of the following best describes how the energy that was in the orange juice actually came from the Sun?

A. The sun provides energy for orange trees to convert carbon dioxide in the air to sugar molecules. The energy is now in the sugar molecules and can then be stored in the fruit of the orange trees. Therefore, the sugar in the oranges comes from the sun.

B. The sun provides the heat for a chemical reaction to take place in the orange trees. This chemical reaction creates ATP in oranges. That ATP energy is transferred to humans when we eat oranges.

C. The sun provides the energy for orange trees to make oranges. The oranges create energy by being squeezed into orange juice. When we drink the juice we can get the energy.

D. Energy from the sun, in combination with the vitamin C in orange juice provide energy for Anna.

Answer: A

Biology Assessment – April 2008 33

Page 34: Ongoing and Integrated - Saginaw Valley State · Web viewOngoing and Integrated B1.1a Generate new questions that can be investigated in the laboratory or field. 1. Which question

B3.1b Illustrate and describe the energy conversions that occur during photosynthesis and respiration.

1. There are many different forms of energy in the world. For living things, energy must be converted to ATP to be useful. Describe the energy conversions that take place to make ATP.

A. Photosynthesis converts energy from carbon dioxide to oxygen. Cellular

respiration converts energy from oxygen to ATP. B. Cellular respiration converts energy from the sun to glucose molecules.

Photosynthesis converts glucose molecules to ATP. C. Cellular respiration converts energy from oxygen to carbon dioxide.

Photosynthesis converts energy from carbon dioxide to ATP. D. Photosynthesis converts energy from the sun to glucose molecules. Cellular

respiration converts glucose molecules to ATP.

Answer: D

B3.1c Recognize the equations for photosynthesis and respiration and identify the reactants and products for both.

1. Choose the appropriate equation for photosynthesis.

A. Carbon dioxide + water + light glucose + oxygen B. Glucose + oxygen ATP + carbon dioxide + waterC. Carbon dioxide + oxygen + light glucose + carbon dioxide D. None of the above.

Answer: A

2. Below are all the reactants and products of respiration. Identify the reactants and the products.

Carbon dioxide, oxygen, ATP, glucose, water

A. Reactants = water, ATP Products = carbon dioxide, oxygen, glucose B. Reactants = oxygen, glucose Products = ATP, carbon dioxide, water C. Reactants = carbon dioxide, glucose Products = ATP, oxygen, water D. Reactants = oxygen, ATP, water Products = carbon dioxide, glucose

Answer: B

Biology Assessment – April 2008 34

Page 35: Ongoing and Integrated - Saginaw Valley State · Web viewOngoing and Integrated B1.1a Generate new questions that can be investigated in the laboratory or field. 1. Which question

B3.1d Explain how living organisms gain and use mass through the processes of photosynthesis and respiration.

1. Kyle had a plant seed that weighed 0.5 grams and he had 25 grams of dry soil. He wanted to see how big the seed could get if he watered it every day for 2 months. He planted the seed in the soil and gave it the same amount of water each day. After 2 months the seed had grown into a plant 2 feet tall. He then dried and weighed the plant and soil again. The plant now had a mass of 8.5 grams and the soil stilled had a mass of 25 grams. Where did most of the plant’s mass come from?

A. The water provides all of the mass for the plant. B. Minerals in the soil allow the plant to gain mass. C. Carbon dioxide in the air. D. Sunlight.

Answer: C

2. Humans are continuously gaining and losing weight. Explain how, in terms of respiration, you could lose 10 pounds if you cut back calories and got on an exercise program.

A. When you exercise your cells are in need of more ATP to power cellular activity. Cellular respiration can make ATP by breaking down food molecules or fat stored in your body. If you limit your food intake then your fat storage will be used to power respiration.

B. You lose more weight when you exercise and cut back on food intake because the water weight from the food is sweated out when you’re working out.

C. Respiration is used to create food energy for your cells to use when you are working out. Therefore you need less food intake and you can lose weight.

D. None of the above.

Answer: A

B3.1e Write the chemical equation for photosynthesis and cellular respiration and explain in words what they mean.

1. Pick out the correct equation for photosynthesis.

A. 6CO2 + 6H2O + light C6H12O6 + 6O2

B. C6H12O6 + 6O2 light + 6H2O + 6CO2 C. 6CO2 + light C6H12O6 + 6O2

D. None of the above

Answer: A

Biology Assessment – April 2008 35

Page 36: Ongoing and Integrated - Saginaw Valley State · Web viewOngoing and Integrated B1.1a Generate new questions that can be investigated in the laboratory or field. 1. Which question

2. Explain in words what the symbols mean in this equation. 6CO2 + 6H2O + light C6H12O6 + 6O2

A. Glucose plus oxygen with the energy from light make carbon dioxide and water. B. Carbon dioxide plus water with the energy from light make glucose and oxygen. C. Carbon monoxide plus water make glucose plus carbon dioxide D. None of the above

Answer: B

B3.2 Ecosystems The chemical elements that make up the molecules of living things pass through food webs and are combined and recombined in different ways. At each link in an ecosystem, some energy is stored in newly made structures, but much is dissipated into the environment as heat. Continual input of energy from sunlight keeps the process going.

B3.2a Identify how energy is stored in an ecosystem.

1. Which one of the following is an example of why hamburger is stored energy?

A. Hamburger is not stored energy. B. Hamburger is stored energy because it contains molecules that have high energy

bonds. C. Hamburger is stored energy because energy comes from eating producers. D. Both B and C are correct.

Answer: C

B3.2b Describe energy transfer through an ecosystem, accounting for energy lost to the environment as heat.

1. Energy is transferred through ecosystems from one trophic level to the next by organisms eating one another. Unfortunately, not all of the energy is available for transfer to the next trophic level. Why does this happen?

A. Organisms store most of the energy in a place where predators can’t get it. B. Most of the energy is given off as light. C. The organisms use most of the energy for electricity. D. Organisms use the energy and a good portion is given off as heat.

Answer: D

Biology Assessment – April 2008 36

Page 37: Ongoing and Integrated - Saginaw Valley State · Web viewOngoing and Integrated B1.1a Generate new questions that can be investigated in the laboratory or field. 1. Which question

B3.2c Draw the flow of energy through an ecosystem. Predict changes in the food web when one or more organisms are removed.

1. If a single plant species is removed from a food web, then most likely –

A. an animal species will fill the unoccupied nicheB. other plants will produce enough food for herbivoresC. dependent herbivores will have to find new food sourcesD. carnivores will be unaffected by the loss

Answer: C

The next two questions should be answered using the following examples.

A. Grass Cow HumanB. Grass Cow HumanC. Grass Human Cow D. Grass Human Cow

1. Which of the following above is the best example of showing the flow of energy through an ecosystem?

A. AB. BC. CD. D

Answer: A

2. What changes would occur in this food chain if there was a drought and all of the grass died?

A. No change would occurB. Both the cows and the humans would have less food available C. The number of cows would decreaseD. Only the number of humans would decrease

Answer: B or C

B3.3b Describe environmental processes (e.g., the carbon and nitrogen cycles) and their role in processing matter crucial for sustaining life.

1. Which of the following is the best example of an organism maintaining homeostasis?

A. a wolf panting after a chaseB. a spider catching an insect in a webC. a cricket becoming infected by a virusD. a mole digging tunnels in the ground

Answer: A

Biology Assessment – April 2008 37

Page 38: Ongoing and Integrated - Saginaw Valley State · Web viewOngoing and Integrated B1.1a Generate new questions that can be investigated in the laboratory or field. 1. Which question

2. The diagram shows several phases of the nitrogen cycle. Which of the following describes the most likely effect of removing some plants from the area by using chemical herbicides?

A. The rate of erosion of rocks on the ground would be slowed. B. The flow of necessary nutrients would be disrupted. C. The ability of plants to complete photosynthesis would be increased. D. The infiltration of water into the ground would be halted.

Answer: B

3.  Where does water in a lake get most of its energy to evaporate? 

A. The sun heating the lake B. Green plants living in the lake C. Streams entering the lake D. Cold springs under the lake

Answer: A

Biology Assessment – April 2008 38

Page 39: Ongoing and Integrated - Saginaw Valley State · Web viewOngoing and Integrated B1.1a Generate new questions that can be investigated in the laboratory or field. 1. Which question

B3.3a Use a food web to identify and distinguish producers, consumers, and decomposers and explain the transfer of energy through trophic levels.

How One Fungus Get Nutrients

1. Which word best describes the fungus in the situation above?

A. Predator B. Producer C. Parasite D. Decomposer

Answer: A

2. About 10% of the energy at one trophic level is passed to the next level. What usually happens to the energy that is not passed to the next trophic level or used to carry out life processes?

A. It is given off as heat. B. It is stored as vitamins. C. It is used in reproduction. D. It is used in protein synthesis.

Answer: A

Biology Assessment – April 2008 39

A certain type of fungus grows sticky structures when roundworms are near.

Roundworms that come close to the sticky structures can become trapped.

The fungus penetrates and

Page 40: Ongoing and Integrated - Saginaw Valley State · Web viewOngoing and Integrated B1.1a Generate new questions that can be investigated in the laboratory or field. 1. Which question

The Collared Peccary

3. Which of these is a food web based on the relationships described above?

A. B.

C. D.

Answer: B

Biology Assessment – April 2008 40

The collared peccary is often mistaken for a pig. It has a short pig-like snout and crushing molars. The peccary has a mass of 10-20 kg and a shoulder height of about 50 cm.

The collared peccary is omnivorous, eating most roots, seeds, fruit, cacti, and occasionally insects and mice. Peccaries travel in herds of 6-30 animals. The natural enemies of the peccary are bobcats and coyotes.

Page 41: Ongoing and Integrated - Saginaw Valley State · Web viewOngoing and Integrated B1.1a Generate new questions that can be investigated in the laboratory or field. 1. Which question

4. The diagram above shows a food web in a large park. Each circle represents a different species in the food web. Which of the organisms in the food web could be referred to as primary consumers?

  A. 7 only B.  5 and 6 only C.  2, 3, and 4 only D.  2, 5, and 7 only

Answer: C

B3.3 Element Recombination As matter cycles and energy flows through different levels of organization of living systems—cells, organs, organisms, and communities—and between living systems and the physical environment, chemical elements are recombined in different ways. Each recombination results in storage and dissipation of energy into the environment as heat. Matter and energy are conserved in each change.

Biology Assessment – April 2008 41

Page 42: Ongoing and Integrated - Saginaw Valley State · Web viewOngoing and Integrated B1.1a Generate new questions that can be investigated in the laboratory or field. 1. Which question

Unit 8: Population Ecology & Human Impacts on EcosystemsTime: 2 weeks

B2.2g Propose how moving an organism to a new environment may influence its ability to survive and predict the possible impact of this type of transfer.

1. Predict what you think would happen to a small population of ground squirrels from Michigan that were moved to the Arizona Desert.

A. The population of the squirrels would increase because they would have no natural predators.

B. The population of the squirrels would decrease because there are not enough available resources (water and food).

C. The population of squirrels would increase because there would be more available food.

D. The population of squirrels would decrease because snakes would eat them all.

Answer: B

B3.4 Changes in Ecosystem Although the interrelationships and interdependence of organisms may generate biological communities in ecosystems that are stable for hundreds or thousands of years, ecosystems always change when climate changes or when one or more new species appear as a result of migration or local evolution. The impact of the human species has major consequences for other species.

B3.4a Describe ecosystem stability. Understand that if a disaster such as flood or fire occurs, the damaged ecosystem is likely to recover in stages of succession that eventually result in a system similar to the original one.

1. Which of the following examples do you think would happen to a temperate forest ecosystem if a fire destroyed most of the vegetation?

A. The ecosystem would recover in predictable stages until it resulted in a similar or more mature forest.

B. The ecosystem would never be able to grow again. C. The ecosystem would grow, but only grass would ever be able to grow there

again. D. The ecosystem would immediately grow trees again.

Answer: A

Biology Assessment – April 2008 42

Page 43: Ongoing and Integrated - Saginaw Valley State · Web viewOngoing and Integrated B1.1a Generate new questions that can be investigated in the laboratory or field. 1. Which question

B3.4c Examine the negative impact of human activities.

1. Which of the following do you think is the most harmful human activity on the environment?

A. Recycling B. Smoking C. Deforestation D. Nature trails

Answer: C

2. What ecological problem will result if individuals use automobiles instead of car-pooling or using scheduled public transportation?

A. convenience in traveling when and where each individual chooses. B. greater air pollution than from any other cause. C. higher employment in automobile and fuel industries. D. lower accident and injury rate than in scheduled transportation.

Answer: B

B3.4x Human Impact Humans can have tremendous impact on the environment. Sometimes their impact is beneficial, and sometimes it is detrimental.

B3.4d Describe the greenhouse effect and list possible causes.

1. How are gases in the atmosphere, such as carbon dioxide, like the glass windows of a greenhouse?

A. They are both made up of moleculesB. They are both transparent C. They both allow sunlight in and trap heat in the area D. They are not similar at all

Answer: C

Biology Assessment – April 2008 43

Page 44: Ongoing and Integrated - Saginaw Valley State · Web viewOngoing and Integrated B1.1a Generate new questions that can be investigated in the laboratory or field. 1. Which question

B3.4e List the possible causes and consequences of global warming.

1. How are greenhouse gases and global warming interconnected?

A. Greenhouse gases make it ridiculously hot and human and they make it constantly hot where they are located.

B. Increasing greenhouse gases can slightly increase the temperature of earth’s atmosphere.

C. Decreasing greenhouse gases can increase the temperature of the earth significantly.

D. There is no connection between these topics.

Answer: B 2. Which of the following would help to reduce the greenhouse effect the MOST?

A. Decreasing car emissionB. Planting more treesC. Stop the use of spray cansD. Reducing cow ranches

 Answer: A

3. Which of the following governmental regulations would NOT help reduce global warming?

A. Reducing the movement of people to coastal cities.B. Increasing the miles per gallon requirements for cars and SUVs.C. Mandating recycling in all areas.D. Reforestation and sustained-tree farming.

 Answer: A

4. What chemical compound seems to be the MOST responsible for global warming?

A. MethaneB. ChlorofluorocarbonsC. Carbon DioxideD. Sodium Chloride

 Answer: C

5. How could the global warming best be studied?

A. Measure the amount of carbon dioxide in the air in one area.B. Measure the temperatures around the world for several years.C. Measure the thickness of the polar ice caps in the Arctic.D. Measure the amount of methane produced by a cow in one day.

Answer: B

Biology Assessment – April 2008 44

Page 45: Ongoing and Integrated - Saginaw Valley State · Web viewOngoing and Integrated B1.1a Generate new questions that can be investigated in the laboratory or field. 1. Which question

6. What ecological problem will result if individuals use automobiles instead of car-pooling or using scheduled public transportation?

A. convenience in traveling when and where each individual chooses. B. greater air pollution than from any other cause. C. higher employment in automobile and fuel industries. D. lower accident and injury rate than in scheduled transportation.

Answer: B

B3.5 Populations Populations of living things increase and decrease in size as they interact with other populations and with the environment. The rate of change is dependent upon relative birth and death rates.

Biology Assessment – April 2008 45

Page 46: Ongoing and Integrated - Saginaw Valley State · Web viewOngoing and Integrated B1.1a Generate new questions that can be investigated in the laboratory or field. 1. Which question

Use the information in the passage and the graph below to answer the following question.

Northwest of Michigan’s Upper Peninsula, in Lake Superior, is Isle Royale National Park. The island is 45 miles long and 5 to 8 miles wide. In the early 1900s, moose arrived on the island (apparently swimming over from Canada). With no predators on the island, and plenty of water plants, twigs, and leaves for the moose to feed on, the moose population increased greatly. By the 1930s, the food was depleted, and hundreds of moose starved. Following a fire, the food supply—and the moose—recovered. About 10 years later the food dwindled again, and the moose starved once more.

During 1948-1949, as a result of an extremely cold winter, Lake Superior froze over. A pack of eastern timber wolves migrated across the ice from Canada and established themselves on the island. The wolf population now numbers between 25 and 40. The wolves kill the very young, very old, sick or weak moose. The moose population, at a ratio of about 30 moose per wolf, is now stable and healthy. The balance is aided by a population of beaver, which builds dams for ponds and beaver meadows, supplying the moose with additional plant growth.

1. How do the moose benefit from the beaver population?

A. The beaver increase the water supply by building dams.B. The beaver create an environment favorable to plant growth.C. The beaver have no effect on the survival of the moose.D. The beaver are an alternative food source for the moose.

Answer: B

Biology Assessment – April 2008 46

Page 47: Ongoing and Integrated - Saginaw Valley State · Web viewOngoing and Integrated B1.1a Generate new questions that can be investigated in the laboratory or field. 1. Which question

B3.5a Graph changes in population growth, given a data table.

1. If the population increases by the same rate from the year 1990 to the year 2000 as in the years from 1980 to 1990, approximately what is the expected population by the year 2000?

A. 47 millionB. 50 millionC. 53 millionD. 58 million

Answer: C

B3.5c Predict the consequences of an invading organism on the survival of other organisms.

1. Predict what you think would happen to grasses and other vegetation if we moved a great deal of gazelle from Africa to Florida.

A. The grasses would not be affected because the alligators would eat all of the gazelle.

B. The grasses would decrease in numbers because the gazelles feed on the grass. C. The grasses would decrease in numbers because gazelles carry diseases. D. None of the above.

Answer: B

B3.5f Graph an example of exponential growth. Then show the population leveling off at the carrying capacity of the environment.

1. Why does the white-tailed deer population drop when the carrying capacity is exceeded?

A. Resources are too low to support the population.

B. Weather changes reduce the deer population.

C. The height of edible plants exceeds the height of the deer.

D. Competition by other animals is greatly reduced.

Answer: A

Biology Assessment – April 2008 47

Page 48: Ongoing and Integrated - Saginaw Valley State · Web viewOngoing and Integrated B1.1a Generate new questions that can be investigated in the laboratory or field. 1. Which question

B3.5e Recognize that and describe how the physical or chemical environment may influence the rate, extent, and nature of population dynamics within ecosystems.

1. If carbon dioxide (CO2) were withdrawn from the biosphere, which organism would first experience negative biological effects?

A. Primary consumers B. Producers C. Second-level consumers D. Third-level consumers

Answer: B

2. Water has always been scarce in the Middle East. Israel, Jordan, Syria, the West Bank, Iraq and Turkey have fought over water and continually threaten war when the flow of water is reduced. Where the flow of water has slowed down, we see problems obtaining clean drinking water, difficulty growing crops, and problems in producing electricity. Where water is plentiful, we see large economic growth of Middle Eastern cities.

 How does this scenario describe water?

A. VariableB. Biotic componentC. ProducerD. Abiotic factor

Answer: D

3. Which of these DECREASES as the number of fish in a small pond increases?

A. Competition for foodB. Levels of body wastes in the pondC. Number of available nesting sitesD. Number of predators

Answer: C

B3.5x Environmental Factors The shape of population growth curves vary with the type of organism and environmental conditions, such as availability of nutrients and space. As the population increases and resources become scarcer, the population usually stabilizes at the carrying capacity of that environment.

Biology Assessment – April 2008 48

Page 49: Ongoing and Integrated - Saginaw Valley State · Web viewOngoing and Integrated B1.1a Generate new questions that can be investigated in the laboratory or field. 1. Which question

Unit 9: Cell Division & Chromosome MutationTime: 3 weeks

B4.2x DNA, RNA, and Protein Synthesis Protein synthesis begins with the information in a sequence of DNA bases being copied onto messenger RNA. This molecule moves from the nucleus to the ribosome in the cytoplasm where it is “read.” Transfer RNA brings amino acids to the ribosome, where they are connected in the correct sequence to form a specific protein.

B4.2f Demonstrate how the genetic information in DNA molecules provides instructions for assembling protein molecules and that this is virtually the same mechanism for all life forms.

1. Although there are a limited number of amino acids, many different types of proteins exist because the

A. size of a given amino acid can vary.B. chemical composition of a given amino acid can vary.C. sequence and number of amino acids is different.D. same amino acid can have many different properties.

Answer: C

B4.2g Describe the processes of replication, transcription, and translation and how they relate to each other in molecular biology.

1. A strand of mRNA containing the repeating sequence AAGAAGAAGAAG could code for which of the following amino acid sequences?

A. lys-arg-glu-lysB. ser-ser-glu-gluC. lys-arg-lys-argD. lys-lys-lys-lys

Answer: D

Biology Assessment – April 2008 49

Page 50: Ongoing and Integrated - Saginaw Valley State · Web viewOngoing and Integrated B1.1a Generate new questions that can be investigated in the laboratory or field. 1. Which question

2. This illustration is an example of a normal DNA sequence. Which of the following represents a single base change in the sequence?

A. B.

C. D.

Answer: D

B4.4x Genetic Variation Genetic variation is essential to biodiversity and the stability of a population. Genetic variation is ensured by the formation of gametes and their combination to form a zygote. Opportunities for genetic variation also occur during cell division when chromosomes exchange genetic material causing permanent changes in the DNA sequences of the chromosomes. Random mutations in DNA structure caused by the environment are another source of genetic variation.

B4.4c Explain how mutations in the DNA sequence of a gene may be silent or result in phenotypic change in an organism and in its offspring.

1. Mutations within a DNA sequence are

A. natural processes that produce genetic diversity.B. natural processes that always affect the phenotype.C. unnatural processes that always affect the phenotype.D. unnatural processes that are harmful to genetic diversity.

Answer: A

B2.1 Transformation of Matter and Energy in Cells In multicellular organisms, cells are specialized to carry out specific functions such as transport, reproduction, or energy transformation.

B2.1c Explain cell division, growth, and development as a consequence of an increase in cell number, cell size, and/or cell products.

Biology Assessment – April 2008 50

Page 51: Ongoing and Integrated - Saginaw Valley State · Web viewOngoing and Integrated B1.1a Generate new questions that can be investigated in the laboratory or field. 1. Which question

B2.1x Cell Differentiation Following fertilization, cell division produces a small cluster of cells that then differentiate by appearance and function to form the basic tissues of an embryo.

B2.1d Describe how, through cell division, cells can become specialized for specific function.

1. The information in the box identifies some of the organs of the kitten. Which of the following is identical for every cell in each of the four organs?

A. Amount of ATP B. Function of cell C. Size of cells D. Genes in DNA

Answer: D

2.   Biologists know that nearly all cells in a person’s body contain the same genes. For example, kidney cells contain the same genes as the cells that normally make hemoglobin. Given these facts, explain why kidney cells do not make hemoglobin even though they contain the hemoglobin gene.

B3.5 Populations Populations of living things increase and decrease in size as they interact with other populations and with the environment. The rate of change is dependent upon relative birth and death rates.

B3.5d Describe different reproductive strategies employed by various organisms and explain their advantages and disadvantages.

1. Which of the following is a form of sexual reproduction?

A. A sponge forming from a piece of the parent. B. A yeast cell undergoing budding. C. A raspberry bush growing from a piece of buried stem. D. A rose flower producing seeds.

Answer: D

Biology Assessment – April 2008 51

Page 52: Ongoing and Integrated - Saginaw Valley State · Web viewOngoing and Integrated B1.1a Generate new questions that can be investigated in the laboratory or field. 1. Which question

B4.2 DNA The genetic information encoded in DNA molecules provides instructions for assembling protein molecules. Genes are segments of DNA molecules. Inserting, deleting, or substituting DNA segments can alter genes. An altered gene may be passed on to every cell that develops from it. The resulting features may help, harm, or have little or no effect on the offspring’s success in its environment.

B4.2a Show that when mutations occur in sex cells, they can be passed on to offspring (inherited mutations), but if they occur in other cells, they can be passed on to descendant cells only (non-inherited mutations).

1. How can radiation produce mutations in the offspring of an exposed organism?

A. It doubles the amount of DNA within egg or sperm cells.B. It changes recessive traits to dominant traits and vice versa.C. It allows non-DNA proteins to control the expression of various traits.D. It rearranges the genetic information in the DNA of an egg or sperm cell.

Answer: D

B4.3 Cell Division- Mitosis and Meiosis Sorting and recombination of genes in sexual reproduction results in a great variety of possible gene combinations from the offspring of any two parents.

B4.3a Compare and contrast the processes of cell division (mitosis and meiosis), particularly as those processes relate to production of new cells and to passing on genetic information between generations.

1. The diagram below shows homologous chromosomes during prophase I of meiosis. Which of the following correctly describes the process being illustrated?

A. mutation in which the DNA content of the gene is altered

B. segregation of sister chromatidsC. condensation and segregation of allelesD. crossing-over in which alleles are exchanged

Answer: D

Biology Assessment – April 2008 52

Page 53: Ongoing and Integrated - Saginaw Valley State · Web viewOngoing and Integrated B1.1a Generate new questions that can be investigated in the laboratory or field. 1. Which question

2. Which of the following best describes the formation of a zygote?

A. A sperm cell nucleus and an egg cell nucleus fuse.B. A cell’s DNA replication and mitosis are accelerated.C. A succession of cell divisions produces a solid mass of cells.D. A cell with 46 chromosomes divides to form cells with 23 chromosomes each.

Answer: A

3. The diagram below provides information about a carrot cell.

A carrot cell contains 18 chromosomes. Which of the following diagrams illustrates the correct number of chromosomes in new cells produced by mitosis?

A. B.

C. D.

Answer: B

4. The body cells of an individual plant have 50 chromosomes. How many chromosomes would be found in the gametes produced this plant?

A. 5B. 10C. 25D. 50

Answer: C

Biology Assessment – April 2008 53

Page 54: Ongoing and Integrated - Saginaw Valley State · Web viewOngoing and Integrated B1.1a Generate new questions that can be investigated in the laboratory or field. 1. Which question

B4.3b Explain why only mutations occurring in gametes (sex cells) can be passed on to offspring.

1. How can radiation produce mutations in the offspring of an exposed organism?

A. It doubles the amount of DNA within egg or sperm cells.B. It changes recessive traits to dominant traits and vice versa.C. It allows non-DNA proteins to control the expression of various traits.D. It rearranges the genetic information in the DNA of an egg or sperm cell.

Answer: D

B4.3c Explain how it might be possible to identify genetic defects from just a karyotype of a few cells.

1. You are a genetic counselor and a very distraught couple comes into your office with a child who has cystic fibrosis. The husband accuses the wife "You caused this! It's well known that mothers who drink have sick babies!" What would be the most accurate information you could give this couple?

 A. We don’t know how this disease is caused, but research is making progress every

day. B. It is our theory that this disease is based on many factors that are too complicated

to understand. C. Cystic fibrosis is caused by both parents who contribute a faulty gene. D. The facts of science are too complicated and you don't have the training to

comprehend it clearly.

Answer: C

Biology Assessment – April 2008 54

Page 55: Ongoing and Integrated - Saginaw Valley State · Web viewOngoing and Integrated B1.1a Generate new questions that can be investigated in the laboratory or field. 1. Which question

Karyotypes

2. Compare the two karyotypes. Where are they different?

A. 23rd pairB. 21st pair C. 5th pair D. 11th pair

Answer: A

3. These karyotypes are of:

A. 2 femalesB. 2 males C. 1 female, 1 male D. You can't tell from the information

 Answer: C

4. Which chromosome is abnormal or carries a genetic mutation?

A. 23rd pairB. 21st pair C. 18th pair D. 13th pair

 Answer: B

Biology Assessment – April 2008 55

Page 56: Ongoing and Integrated - Saginaw Valley State · Web viewOngoing and Integrated B1.1a Generate new questions that can be investigated in the laboratory or field. 1. Which question

5. What is this genetic condition known as?

A. Trisomy 18B. Down Syndrome C. Trisomy 21 D. Answers B & D

  Answer: D

6. The general characteristics of this genetic condition are

A. broad chest, webbed neck skin, short statureB. underdeveloped sex organs, heart impairment C. mental retardation, rounded faces, almond-shaped eyes D. aggressive behavior, tall, thin

Answer: C

B4.3d Explain that the sorting and recombination of genes in sexual reproduction result in a great variety of possible gene combinations from the offspring of two parents.

B4.3e Recognize that genetic variation can occur from such processes as crossing over, jumping genes, and deletion and duplication of genes.

1. Which of the following would be an example of a frameshift mutation?

A. The base guanine is changed to adenine.B. A single base is deleted from DNA causing mRNA to be out of position by one C. Homologous chromosomes fail to separate properly.D. Part of a chromosome breaks out and is reinserted backwards.

Answer: B

2. Which of these would most likely cause a mutation?

A. the placement of ribosomes on the endoplasmic reticulum B. the insertion of a nucleotide into DNAC. the movement of transfer RNA out of the nucleusD. the release of messenger RNA from DNA

Answer: B

Biology Assessment – April 2008 56

Page 57: Ongoing and Integrated - Saginaw Valley State · Web viewOngoing and Integrated B1.1a Generate new questions that can be investigated in the laboratory or field. 1. Which question

B4.3f Predict how mutations may be transferred to progeny.

1. How can radiation produce mutations in the offspring of an exposed organism?

A. It doubles the amount of DNA within egg or sperm cells.B. It changes recessive traits to dominant traits and vice versa.C. It allows non-DNA proteins to control the expression of various traits.D. It rearranges the genetic information in the DNA of an egg or sperm cell.

Answer: D

B4.4b Explain that gene mutation in a cell can result in uncontrolled cell division called cancer. Also know that exposure of cells to certain chemicals and radiation increases mutations and thus increases the chance of cancer.

1. The p53 gene helps prevent individual cells from becoming cancerous. One of the techniques employed by the p53 gene is apoptosis, which means

A. repairing mutated DNAB. programmed cell deathC. stopping the cell cycleD. enhancing meiosis

Answer: B

B4.4x Genetic Variation Genetic variation is essential to biodiversity and the stability of a population. Genetic variation is ensured by the formation of gametes and their combination to form a zygote. Opportunities for genetic variation also occur during cell division when chromosomes exchange genetic material causing permanent changes in the DNA sequences of the chromosomes. Random mutations in DNA structure caused by the environment are another source of genetic variation.

Biology Assessment – April 2008 57

Page 58: Ongoing and Integrated - Saginaw Valley State · Web viewOngoing and Integrated B1.1a Generate new questions that can be investigated in the laboratory or field. 1. Which question

Unit 10: DNA/RNA and Protein SynthesisTime: 4 weeks

B4.2x DNA, RNA, and Protein Synthesis Protein synthesis begins with the information in a sequence of DNA bases being copied onto messenger RNA. This molecule moves from the nucleus to the ribosome in the cytoplasm where it is “read.” Transfer RNA brings amino acids to the ribosome, where they are connected in the correct sequence to form a specific protein.

B4.2h Recognize that genetic engineering techniques provide great potential and responsibilities.

1. Genetic engineering has produced goats whose milk contains proteins that can be used as medicines. This effect was produced by

A. mixing foreign genes into the milk.B. injecting foreign genes into the goats’ udders.C. inserting foreign genes into fertilized goat eggs. D. genetically modifying the nutritional needs of the goats’ offspring.

Answer: C

2.  Which of the following is an example of genetic engineering? 

A.  Growing a whole plant from a single cell.B.  Finding the sequences of bases in plant DNA.C.  Inserting a gene into plants that makes them resistant to insects.D.  Attaching the root of one type of plant to the stem of another type of plant.

 Answer: C

B4.4x Genetic Variation Genetic variation is essential to biodiversity and the stability of a population. Genetic variation is ensured by the formation of gametes and their combination to form a zygote. Opportunities for genetic variation also occur during cell division when chromosomes exchange genetic material causing permanent changes in the DNA sequences of the chromosomes. Random mutations in DNA structure caused by the environment are another source of genetic variation.

1.  Animals that reproduce sexually differ from animals that reproduce asexually in that sexually reproducing animals have

  A.  a larger number of offspringB.  more genetic variation among their offspringC.  offspring that are nearly identical to their parentsD.  offspring that are perfectly adapted to their parents' habitat

Answer: B

Biology Assessment – April 2008 58

Page 59: Ongoing and Integrated - Saginaw Valley State · Web viewOngoing and Integrated B1.1a Generate new questions that can be investigated in the laboratory or field. 1. Which question

B4.4a Describe how inserting, deleting, or substituting DNA segments can alter a gene. Recognize that an altered gene may be passed on to every cell that develops from it and that the resulting features may help, harm, or have little of no effect on the offspring’s success in its environment.

1. Which of these would MOST likely cause a mutation?

A. the placement of ribosomes on the endoplasmic reticulumB. the insertion of a nucleotide into DNAC. the movement of transfer RNA out of the nucleusD. the release of messenger RNA from DNA

Answer: B

2. Which of these best explains how mutation can be beneficial to an organism?

A. Phenotypic change may create an advantage over other organisms. B. Recombined genetic material improves genotype stability. C. Mitosis becomes a favored means of reproduction. D. Deoxyribose sugars develop into additional nucleotides.

Answer: A

B4.1 Genetics and Inherited Traits Hereditary information is contained in genes, located in the chromosomes of each cell. Cells contain many thousands of different genes. One or many genes can determine an inherited trait of an individual, and a single gene can influence more than one trait. Before a cell divides, this genetic information must be copied and apportioned evenly into the daughter cells.

B4.1b Explain that the information passed from parents to offspring is transmitted by means of genes that are coded in DNA molecules. These genes contain the information for the production of proteins.

1. Genetic information is contained in the DNA of an organism. How does DNA code for specific proteins that are unique in each individual?

A. DNA codes for RNA which codes for amino acids which hook together to make proteins.

B. DNA makes amino acids which codes for RNA thus gives proteins.C. DNA codes RNA which gives proteins.D. DNA codes for amino acids which code for proteins.

Answer: A

Biology Assessment – April 2008 59

Page 60: Ongoing and Integrated - Saginaw Valley State · Web viewOngoing and Integrated B1.1a Generate new questions that can be investigated in the laboratory or field. 1. Which question

B4.2 DNA. The genetic information encoded in DNA molecules provides instructions for assembling protein molecules. Genes are segments of DNA molecules. Inserting, deleting, or substituting DNA segments can alter genes. An altered gene may be passed on to every cell that develops from it. The resulting features may help, harm, or have little or no effect on the offspring’s success in its environment.

1. The bacterium Agrobacterium tumefaciens infects plants, and a portion of its DNA is inserted into the plant's chromosomes. This causes the plant to produce gall cells, which manufacture amino acids that the bacterium uses as food. This process is a natural example of

A. polyploidy.B. genetic manipulation.C. grafting.D. hybridization.

Answer: B

B4.2b Recognize that every species has its own characteristic DNA sequence.

1. Although there are a limited number of amino acids, many different types of proteins exist because the

A. size of a given amino acid can vary.B. chemical composition of a given amino acid can vary.C. sequence and number of amino acids is different.D. same amino acid can have many different properties.

Answer: C

B4.2c Describe the structure and function of DNA.

1. Erwin Chargaff studied the DNA of organisms within a single species. Chargaff discovered that the amount of adenine is about equal to the amount of thymine. Which of these explains why the ratio of adenine to thymine is nearly 1:1?

A. Adenine and thymine pair with each other. B. Adenine binds with phosphates, while thymine binds with nitrates. C. Adenine and thymine are identical in chemical composition. D. Adenine bases contain a form of thymine.

Answer: D

Biology Assessment – April 2008 60

Page 61: Ongoing and Integrated - Saginaw Valley State · Web viewOngoing and Integrated B1.1a Generate new questions that can be investigated in the laboratory or field. 1. Which question

2. In all plant and animal cells, the nucleus contains long molecules of DNA. Which of the following best describes the function of DNA?

A. DNA provides the shape and structure of the nucleus. B. DNA packages materials for transport through the nucleus. C. DNA carries materials into and out of the nucleus. D. DNA contains the blueprint for producing the whole organism.

Answer: D

B4.2e Propose possible effects (on the genes) of exposing an organism to radiation and toxic chemicals.

B4.2d Predict the consequences that changes in the DNA composition of particular genes may have on an organism (e.g., sickle cell anemia, other).

1. Sickle-cell anemia is a disorder resulting from a mutation that leads to the production of an abnormal protein. Which component of the DNA molecule provides instructions for the production of the protein?

A. The phosphate groups B. The sugar molecules C. The sequence of nitrogen bases D. The bonds that hold the sugars to the bases

Answer: C

Biology Assessment – April 2008 61

Page 62: Ongoing and Integrated - Saginaw Valley State · Web viewOngoing and Integrated B1.1a Generate new questions that can be investigated in the laboratory or field. 1. Which question

Unit 11: Mendelian and Molecular GeneticsTime: 3 weeks

B4.1 Genetics and Inherited Traits Hereditary information is contained in genes, located in the chromosomes of each cell. Cells contain many thousands of different genes. One or many genes can determine an inherited trait of an individual, and a single gene can influence more than one trait. Before a cell divides, this genetic information must be copied and apportioned evenly into the daughter cells.

1. Katie has red hair, green eyes, and freckles, just like her mother. Her father has brown hair, brown eyes, and no freckles. Why does Katie look more like her mother than she does like her father?

A. because daughters always resemble their mothers more than their fathersB. because Katie's father is older than her motherC. because Katie's father doesn't have any frecklesD. because Katie inherited more traits from her mother than her father

Answer: D

2. A particular species of bird can be brown or white. The white color is a recessive trait, while the brown color is a dominant trait. When two brown birds mate, is it possible for them to have white offspring?

A. No, because both parents have only genes for being brown.B. Yes, because offspring color does not depend on the genes of the parents.C. Yes, because both parents may have and pass on the gene for being white.D. No, because the parents can pass on only the dominant trait to their offspring.

Answer: C

3. In certain breeds of dogs, deafness is due to a recessive allele (d) of a particular gene, and normal hearing is due to its dominant allele (D). What percentage of the offspring of a normal heterozygous (Dd) dog and a deaf dog (dd) would be expected to have normal hearing?

A. 0%B. 25%C. 50%D. 100%

Answer: C

Biology Assessment – April 2008 62

Page 63: Ongoing and Integrated - Saginaw Valley State · Web viewOngoing and Integrated B1.1a Generate new questions that can be investigated in the laboratory or field. 1. Which question

B4.1e Determine the genotype and phenotype of monohybrid crosses using a Punnett Square.

1. If a corn plant has a genotype of Ttyy, what are the possible genetic combinations that could be present in a single grain of pollen from this plant?

A. Ty, tyB. TY, tyC. TY, Ty, tyD. Ty, ty, tY, TY

Answer: A

2. In fruit flies, the gene for red eyes R is dominant and the gene for sepia eyes (r) is recessive. What are the possible genotypes of the offspring of two red-eyed heterozygous flies (Rr)?

A. Red eyes onlyB. sepia eyes onlyC. Rr and rr onlyD. RR, Rr, and rr only

Answer: D

B4.1c Differentiate between dominant, recessive, co-dominant, polygenic, and sex-linked traits.

1. A healthy individual is a carrier of a lethal allele but is unaffected by it. What is the probable genotype of this individual?

A. two dominant normal allelesB. one recessive lethal allele and one dominant lethal alleleC. one recessive lethal allele and one dominant normal alleleD. one dominant lethal allele and one recessive normal allele

Answer: C

B4.1d Explain the genetic basis for Mendel’s laws of segregation and independent assortment.

B4.1a Draw and label a homologous chromosome pair with heterozygous alleles highlighting a particular gene location.

Biology Assessment – April 2008 63

Page 64: Ongoing and Integrated - Saginaw Valley State · Web viewOngoing and Integrated B1.1a Generate new questions that can be investigated in the laboratory or field. 1. Which question

Unit 12: EvolutionTime: 3 weeks

B2.4 Cell Specialization In multicellular organisms, specialized cells perform specialized functions. Organs and organ systems are composed of cells and function to serve the needs of cells for food, air, and waste removal. The way in which cells function is similar in all living organisms.

B2.4a Explain that living things can be classified based on structural, embryological, and molecular (relatedness of DNA sequence) evidence.

1. The human embryo and the embryos of other animals with backbones have gill slits. According to modern evolutionary theory, this best supports the idea that

A. fish are our closest relatives.B. all embryos breathe through gills.C. all animals with backbones are related by common ancestry.D. all organisms have gill slits at an early stage in their development.

Answer: C

2.  According to evolutionary theory, which of the following evolutionary trees best describes the relationship between groups of vertebrates?

A. B. C. D.

Answer: B

Biology Assessment – April 2008 64

Page 65: Ongoing and Integrated - Saginaw Valley State · Web viewOngoing and Integrated B1.1a Generate new questions that can be investigated in the laboratory or field. 1. Which question

B2.4d Analyze the relationships among organisms based on their shared physical, biochemical, genetic, and cellular characteristics and functional processes.

1. Classify each of the eight living things listed below into one of the two groups according to an important physical characteristic.

GorillaParrotSnakeEarthwormJellyfishSpongeFishFly

A. Group 1: Gorilla, Parrott, Snake, & Earthworm Group 2: Jellyfish, Sponge, Fish, & Fly

B. Group 1: Gorilla, Parrott, & FlyGroup 2: Snake, Earthworm, Jellyfish, Sponge, & Fish

C. Group 1: Gorilla, Parrott, Snake, & Fish Group 2: Jellyfish, Sponge, Fly, & EarthwormD. Group 1: Gorilla, Parrott, Snake, Fly, & Earthworm Group 2: Jellyfish, Sponge, & Fish

Answer: C

B3.4 Changes in Ecosystems Although the interrelationships and interdependence of organisms may generate biological communities in ecosystems that are stable for hundreds or thousands of years, ecosystems always change when climate changes or when one or more new species appear as a result of migration or local evolution. The impact of the human species has major consequences for other species.

Biology Assessment – April 2008 65

Page 66: Ongoing and Integrated - Saginaw Valley State · Web viewOngoing and Integrated B1.1a Generate new questions that can be investigated in the laboratory or field. 1. Which question

B3.4b Recognize and describe that a great diversity of species increases the chance that at least some living organisms will survive in the face of cataclysmic changes in the environment.

1. If a cataclysmic event changes an environment with great species diversity the likelihood of some organisms living will increase because

A. a higher level of species diversity is directly related to a higher level of fitness of those organisms.

B. a higher level of species diversity is directly related to the number of organisms.C. a higher level of species diversity is related to the number of niches in the

environment.D. a higher level of species diversity is directly related to a lower level of fitness of

those organisms.

Answer: A

B5.1 Theory of Evolution The theory of evolution provides a scientific explanation for the history of life on Earth as depicted in the fossil record and in the similarities evident within the diversity of existing organisms.

B5.1a Summarize the major concepts of natural selection (differential survival and reproduction of chance inherited variants, depending on environmental conditions).

1.  Which of the following is NOT a part of Darwin's theory of evolution by natural selection? 

A.  Individuals in a population vary in many ways.B.  Some individuals possess features that enable them to survive better than

individuals lacking those features.C.  More offspring are produced than can generally survive.D.  Changes in an individual's genetic material are usually harmful.

Answer: D

2.  Which of the following is most consistent with the modern theory of evolution? 

A.  Parents pass their physical traits to their offspring; those offspring with traits that help them survive in the environment are able to reproduce.

B.  Parents change their physical traits in order to survive in the environment, then those parental traits are passed to their offspring.

C.  Life on this planet came from another planet far out in space.D.  Living organisms have not changed for hundreds of millions of years.

Answer: A

Biology Assessment – April 2008 66

Page 67: Ongoing and Integrated - Saginaw Valley State · Web viewOngoing and Integrated B1.1a Generate new questions that can be investigated in the laboratory or field. 1. Which question

B5.1b Describe how natural selection provides a mechanism for evolution.

1. Which of these best illustrates natural selection?

A. An organism with favorable genetic variations will tend to survive and breed successfully.

B. A population monopolizes all of the resources in its habitat, forcing other species to migrate.

C. A community whose members work together utilizes all existing resources and migratory routes.

D. The largest organisms in a species receive the only breeding opportunities.

Answer: A

2. A man treated his home with a pesticide that kills roaches. The first application of the pesticide killed 92% of the roaches. Two months later he applied the pesticide to his home again, but the second application killed only 65% of the roaches. What would best explain the decrease in the effectiveness of the pesticide?

A. The pesticide is effective only against mature roaches. B. Once roaches learned how to fight the pesticide, they taught others. C. The surviving roaches were naturally resistant to the pesticide, and that resistance

was inherited by their offspring. D. The pesticide caused some of the roaches’ digestive systems to mutate and

metabolize the pesticide.

Answer: C

3. The myxoma virus was used to control an overpopulation of European rabbits in Australia. When first introduced in the mid-1900s, the virus greatly reduced the European rabbit population. Today the virus is not an effective control of the European rabbit population. Fewer European rabbits are affected by the virus today because they have —

A. learned to avoid the virus B. moved away from infected areas C. undergone a change in diet D. developed resistance to the virus

Answer: D

Biology Assessment – April 2008 67

Page 68: Ongoing and Integrated - Saginaw Valley State · Web viewOngoing and Integrated B1.1a Generate new questions that can be investigated in the laboratory or field. 1. Which question

B5.1c Summarize the relationships between present-day organisms and those that inhabited the Earth in the past (e.g., use fossil record, embryonic stages, homologous structures, chemical basis).

1.  Which part of the human skeleton is most like the bones in a bird’s wing?

A. AB. BC. CD. D

Answer: B

B5.1d Explain how a new species or variety originates through the evolutionary process of natural selection.

1. Natural selection can best be described as

A. development of an organism that is better suited for its environment. B. random mutations allowing for organisms that are better suited for their

changing environment. C. development of new species that do not have any advantage over other

species.D. random creation of different looking organisms in similar environments.

Answer: B

Biology Assessment – April 2008 68

Page 69: Ongoing and Integrated - Saginaw Valley State · Web viewOngoing and Integrated B1.1a Generate new questions that can be investigated in the laboratory or field. 1. Which question

B5.1e Explain how natural selection leads to organisms that are well suited for the environment (differential survival and reproduction of chance inherited variants, depending upon environmental conditions).

1. A bat species can hunt perfectly well without its eyes, and yet it still can see. What does this tell us about sight as an evolutionary variation in this species of bat?

A. Sight is an unfavorable trait.B. Sight is a non-inheritable trait.C. Sight may be an unfavorable trait for hunting.D. Sight may be a favorable trait, but not for hunting.

Answer: D

2. A species of finch has been studied on one of the geographically isolated Galapagos Islands for many years. Since the island is small, the lineage of every bird for several generations is known. This allows a family tree of each bird to be developed. Some family groups have survived and others have died out. The groups that survive probably have

A. interbred with other species.B. inherited some advantageous variations.C. found new places on the island to live.D. been attacked by more predators.

Answer: B

3.  A green tree frog lives in a forest. How does the frog's green color help it to survive? 

A.  By helping the frog find other frogsB.  By keeping the frog coolC.  By making the frog hard to see when sitting on leavesD. By allowing the frog to make its own food

Answer: C

4.  The bird shown to the right would be most likely to eat 

A.  insects from the ground B.  nectar from tube-shaped flowers C.  fish from the ocean D.  amphibians from a pond

Answer: B

Biology Assessment – April 2008 69

Page 70: Ongoing and Integrated - Saginaw Valley State · Web viewOngoing and Integrated B1.1a Generate new questions that can be investigated in the laboratory or field. 1. Which question

B5.1f Explain, using examples, how the fossil record, comparative anatomy and other evidence supports the theory of evolution.

1. Late Triassic and Jurassic mammals were small. Most were about the size of a mouse; a few attained domestic cat size. Most were insect eaters or omnivores; a few were probably herbivores. By Cretaceous time, mammals the size of opossums occur in the fossil record; these existed with mouse- sized animals that were the ancestors of living marsupials and placentals. In early Cenozoic time, mammals underwent a tremendous radiation and diversification. (Modified from Novacek, 1994)

The previous statement supports the theory of evolution because

A. it explains how different organisms that lived in different times have similar anatomy and similar fossil records.

B. it explains that current day organisms existed millions of years ago.C. it explains that organisms that lived a million years ago are extinct today.D. it states that mammals underwent a tremendous radiation and diversification.

Answer: A

B5.1g Illustrate how genetic variation is preserved or eliminated from a population through natural selection (evolution) resulting in biodiversity.

1. Male peacocks attract mates by “showing off” their brightly colored, highly ornate tail feathers. When a male peacock is born with an above average plumage (tail feathers) what will happen to the overall population?

A. The unusual male will be reproductively successful and pass on his “unusual genes” resulting in an increased biodiversity.

B. The unusual male will be reproductively successful and pass on his “unusual genes” resulting in decreased biodiversity.

C. The unusual male will be reproductively unsuccessful thus resulting in an increased biodiversity.

D. The unusual male will be reproductively unsuccessful thus resulting in a decreased biodiversity.

Answer: A

B5.2x Molecular Evidence Molecular evidence substantiates the anatomical evidence for evolution and provides additional detail about the sequence in which various lines of descents branched.

Biology Assessment – April 2008 70

Page 71: Ongoing and Integrated - Saginaw Valley State · Web viewOngoing and Integrated B1.1a Generate new questions that can be investigated in the laboratory or field. 1. Which question

B5.2a Describe species as reproductively distinct groups of organisms that can be classified based on morphological, behavioral, and molecular structures.

1. Can a horse and a donkey be considered the same species?

A. Yes, because their behavior is very similar.B. Yes, because their morphology is very similar.C. No, because they do not produce offspring that can reproduce.D. No, because their morphology is different.

Answer: C

B5.2b Explain that the degree of kinship between organisms or species can be estimated from similarity of their DNA and protein sequences.

1. When trying to determine relatedness of two organisms in evolutionary history it is MOST important to determine

A. how similar their bodies look to each other.B. how similar their DNA sequences are to each other.C. where their fossils have been located.D. how similar their environments are.

Answer: B

B5.2c Trace the relationship between environmental changes and changes in the gene pool, such as genetic drift and isolation of subpopulations.

1. A species of finch has been studied on one of the geographically isolated Galapagos Islands for many years. Since the island is small, the lineage of every bird for several generations is known. This allows a family tree of each bird to be developed. Some family groups have survived and others have died out. The groups that survive probably have

A. interbred with other species.B. inherited some advantageous variations.C. found new places on the island to live.D. been attacked by more predators.

Answer: B

Biology Assessment – April 2008 71

Page 72: Ongoing and Integrated - Saginaw Valley State · Web viewOngoing and Integrated B1.1a Generate new questions that can be investigated in the laboratory or field. 1. Which question

2. Which of the following explains this phenomenon?

A. Competition B. Extinction C. Predation D. Speciation

Answer: D

B5.3 Natural Selection Evolution is the consequence of natural selection, the interactions of (1) the potential for a population to increase its numbers, (2) the genetic variability of offspring due to mutation and recombination of genes, (3) a finite supply of the resources required for life, and (4) the ensuing selection from environmental pressure of those organisms better able to survive and leave offspring.

Biology Assessment – April 2008 72

Page 73: Ongoing and Integrated - Saginaw Valley State · Web viewOngoing and Integrated B1.1a Generate new questions that can be investigated in the laboratory or field. 1. Which question

B5.3b Describe the role of geographic isolation in speciation.

1. The Galapagos Island is known for having many organisms that are similar, but considered to be different species. This phenomenon can be explained because

A. the island has been studied for so long that all the species have been discovered.B. species that have been alive for millions of years changing to fit the

environment.C. there is very little human interaction on the island. D. the geographic isolation caused by the island chain lends itself to speciation.

Answer: D

B5.3a Explain how natural selection acts on individuals, but it is populations that evolve. Relate genetic mutations and genetic variety produced by sexual reproduction to diversity within a given population.

1. As the industrial revolution created a change in the environment what happened to the population of moths in the environment?

A. The entire population changed in one generation.B. Individual moths were changed, which created a sudden change in the overall

population.C. Individual moths were affected by the new environment which in turn created

a gradual change in the overall population.D. In individual moths were affected which in turn had no affect on the overall

population.

Answer: C

Biology Assessment – April 2008 73